LSAT and Law School Admissions Forum

Get expert LSAT preparation and law school admissions advice from PowerScore Test Preparation.

 Administrator
PowerScore Staff
  • PowerScore Staff
  • Posts: 8917
  • Joined: Feb 02, 2011
|
#24305
Complete Question Explanation

Weaken. The correct answer choice is (A)

The unsolvable dilemma: Doctors who are too thorough will do unnecessary tests, which are painful and costly; doctors who are careless may miss some serious problem. The author then jumps to an odd conclusion: “It is generally unwise for patients to have medical checkups when they do not feel ill.” Perhaps the unspoken assumption here is that if you don’t feel ill and you visit the doctor, then there’s a risk of unnecessary tests (too much care) but no risk of a missed diagnosis (too little care). Thus, you would be better off just staying home.

We are looking for a single answer choice which would weaken the argument. Perhaps we need a statement which would undermine the unspoken assumption.

Answer choice (A): This is the correct answer choice. If a patient does not feel ill but really is sick, then the patient is taking a huge risk by staying home: he is incurring the risk of a missed diagnosis. Perhaps this risk outweighs the risk of “too much care” which he might incur by visiting the doctor.

Answer choice (B): That physicians are reducing the average time they spend on each checkup suggests that the problem of “too much care” is lessening, and the problem of “too little care” is expanding. However, if somebody is not ill, then the “too little care” issue should not be a problem. So this does not weaken the argument.

Answer choice (C): The recommendation deals with the patient’s decision to visit the doctor or not. This decision rests on how ill the patient feels, not on the patient’s judgment about how much care a doctor should exercise. So this answer choice is irrelevant.

Answer choice (D): If some people cannot afford regular medical checkups, then the recommendation in the stimulus is irrelevant for them. So this answer choice does not have any logical connection with the stimulus.

Answer choice (E): That some physicians get it “just right” does not erase the problem of many physicians exercising too much care or too little care. This is not the best answer choice.

Get the most out of your LSAT Prep Plus subscription.

Analyze and track your performance with our Testing and Analytics Package.